2000 AMC 10 Problems/Problem 20

Revision as of 12:18, 11 January 2009 by Misof (talk | contribs)

Problem

Let $A$, $M$, and $C$ be nonnegative integers such that $A+M+C=10$. What is the maximum value of $A\cdot M\cdot C+A\cdot M+M\cdot C+C\cdot A$?

$\mathrm{(A)}\ 49 \qquad\mathrm{(B)}\ 59 \qquad\mathrm{(C)}\ 69 \qquad\mathrm{(D)}\ 79 \qquad\mathrm{(E)}\ 89$

Solution

The trick is to realize that the sum $AMC+AM+MC+CA$ is similar to the product $(A+1)(M+1)(C+1)$.

If we multiply $(A+1)(M+1)(C+1)$, we get $AMC + AM + AC + MC + A + M + C + 1$.

We know that $(A+M+C)=10$, therefore $(A+1)(M+1)(C+1) = (AMC + AM + AC + MC) + 11$.

Therefore the maximum value of $AMC+AM+MC+CA$ is equal to the maximum value of $(A+1)(M+1)(C+1)-11$. Now we will find this maximum.

Suppose that some two of $A$, $M$, and $C$ differ by at least 2. Then this triple $(A,M,C)$ is surely not optimal. Proof: WLOG let $A\geq C+2$. We can then increase the value of $(A+1)(M+1)(C+1)$ by changing $A\gets A-1$ and $C\gets C+1$.

Therefore the maximum is achieved in the cases where $(A,M,C)$ is a rotation of $(3,3,4)$. The value of $(A+1)(M+1)(C+1)$ in this case is $4\cdot 4\cdot 5=80$. And thus the maximum of $AMC + AM + AC + MC$ is $80-11 = \boxed{69}.


See Also

2000 AMC 10 (ProblemsAnswer KeyResources)
Preceded by
Problem 19
Followed by
Problem 21
1 2 3 4 5 6 7 8 9 10 11 12 13 14 15 16 17 18 19 20 21 22 23 24 25
All AMC 10 Problems and Solutions